Difference between revisions of "2014 AMC 10A Problems/Problem 1"
Bestwillcui1 (talk | contribs) m (→Solution) |
Bestwillcui1 (talk | contribs) m (→Solution) |
||
Line 15: | Line 15: | ||
<cmath>\implies \frac{50}{4}</cmath> | <cmath>\implies \frac{50}{4}</cmath> | ||
Finally, simplifying gives | Finally, simplifying gives | ||
− | < | + | <math></math>\implies \boxed{\textbf{(C)}\ \frac{25}{2}}$ |
(Solution by bestwillcui1) | (Solution by bestwillcui1) | ||
Revision as of 11:07, 9 February 2014
Problem
What is
$\textbf{(A)}\ 3\qquad\textbf{(B)}\ 8\qquad\textbf{(C)}\ \frac{25}{2} \qquad\textbf{(D)}}\ \frac{170}{3}\qquad\textbf{(E)}\ 170$ (Error compiling LaTeX. Unknown error_msg)
Solution
We have Making the denominators equal gives Finally, simplifying gives $$ (Error compiling LaTeX. Unknown error_msg)\implies \boxed{\textbf{(C)}\ \frac{25}{2}}$ (Solution by bestwillcui1)
See Also
2014 AMC 10A (Problems • Answer Key • Resources) | ||
Preceded by Problem 1 |
Followed by Problem 3 | |
1 • 2 • 3 • 4 • 5 • 6 • 7 • 8 • 9 • 10 • 11 • 12 • 13 • 14 • 15 • 16 • 17 • 18 • 19 • 20 • 21 • 22 • 23 • 24 • 25 | ||
All AMC 10 Problems and Solutions |
The problems on this page are copyrighted by the Mathematical Association of America's American Mathematics Competitions.